Last visit was: 28 Apr 2024, 05:32 It is currently 28 Apr 2024, 05:32

Close
GMAT Club Daily Prep
Thank you for using the timer - this advanced tool can estimate your performance and suggest more practice questions. We have subscribed you to Daily Prep Questions via email.

Customized
for You

we will pick new questions that match your level based on your Timer History

Track
Your Progress

every week, we’ll send you an estimated GMAT score based on your performance

Practice
Pays

we will pick new questions that match your level based on your Timer History
Not interested in getting valuable practice questions and articles delivered to your email? No problem, unsubscribe here.
Close
Request Expert Reply
Confirm Cancel
SORT BY:
Date
Tags:
Show Tags
Hide Tags
Intern
Intern
Joined: 28 Feb 2024
Posts: 2
Own Kudos [?]: 4 [4]
Given Kudos: 13
Send PM
Intern
Intern
Joined: 04 Oct 2023
Posts: 41
Own Kudos [?]: 12 [0]
Given Kudos: 24
Location: India
Concentration: Technology, Other
GPA: 3.5
Send PM
Intern
Intern
Joined: 28 Feb 2024
Posts: 2
Own Kudos [?]: 4 [0]
Given Kudos: 13
Send PM
Manager
Manager
Joined: 13 Dec 2023
Status:Applying in R1 of 2024 to t15
Affiliations: University of Tennessee
Posts: 111
Own Kudos [?]: 54 [0]
Given Kudos: 38
Location: United States (CO)
Concentration: Strategy, Finance
GMAT Focus 1:
575 Q76 V81 DI78
GPA: 3.62
WE:Analyst (Consumer Products)
Send PM
Re: Editorial: Our provinceS legislature will soon vote on a measure that [#permalink]
A. It fails to adequately address the possibility that many factors affect provincial budget deficits.


- This says that you can't compare the two cities "apples to apples" so this is the correct answer because it's true.;

B. It confuses a claim about the budget deficit in one province with a similar claim about the deficit in another, neighboring province.

- it doesn't confuse the claim. It clearly mentions that there was a separate city north of them with the same problem that had a budget deficit. If you don't see this then you're not comprehending what the answer choice is telling you.

C. It overlooks the possibility that the budget deficit of the province immediately to the north was higher before that province allowed the union bargaining.

-It clearly states that the city north of them had a higher budget deficit before they enacted the law. So no.

D. It takes for granted that if the measure reduces the budget deficit, then it will not increase wages of government employees.

- That's not what's being asked here and is out of scope. Who cares? If it goes up or down it doesn't weaken the argument or strengthen it.

E. It fails to adequately distinguish a factual claim about the measure's likely consequences from a value judgment that the measure should be enacted.

- This one just tries to confuse you and hopes you pick it because it sounds confusing. I don't know what a value judgement is and I doubt you do either if you're reading this. But the part you should focus on is that it says "it fails to adequately distinguish a factual claim about the consequences." Well that just isn't true. It clearly states what it thinks will happen if it's enacted.
Tutor
Joined: 11 Aug 2023
Posts: 828
Own Kudos [?]: 1429 [2]
Given Kudos: 75
GMAT 1: 800 Q51 V51
Send PM
Editorial: Our provinceS legislature will soon vote on a measure that [#permalink]
2
Kudos
Expert Reply
­­Editorial: Our province's legislature will soon vote on a measure that would allow government employee unions to bargain over wages. Some critics claim this measure would increase wages and thus exacerbate our province's budget deficit. But in fact, it would probably reduce the deficit. The province immediately north of ours allows such union bargaining, and its budget deficit is lower than our province's—and lower than it was before the province allowed union bargaining.

The conclusion of the argument is the following:

it (a measure that would allow government employee unions to bargain over wages) would probably reduce the deficit

The support for the conclusion is the following:

The province immediately north of ours allows such union bargaining, and its budget deficit is lower than our province's—and lower than it was before the province allowed union bargaining.

So, the argument uses the fact that another province allows union bargaining and has a budget deficit that is lower than that of "our province" and is lower than it was before union bargaining to support the conclusion that the budget deficit of "our province" would probably decrease as a result of allowing union bargaining.

The editorial's argument is most vulnerable to criticism on which of the following grounds? 

This is a Flaw question, and the correct answer will describe a flaw in the argument.

A. It fails to adequately address the possibility that many factors affect provincial budget deficits.

The argument does exactly what this choice describes.

Let's review the evidence the argument uses to support the conclusion that the measure would probably reduce the deficit.

The province immediately north of ours allows such union bargaining, and its budget deficit is lower than our province's

This premise provides some support for the conclusion. At the same time, there could be many other factors that underlie the difference in budget deficits. So, the cause of the difference may not be that one province allows union bargaining whereas the other doesn't.

The province immediately north of ours allows such union bargaining, and its budget deficit is ...and lower than it was before the province allowed union bargaining.

Even this outcome could be the result of many factors other than allowing union bargaining.

In general, the fact that a province allows union bargaining and has a lower deficit doesn't necessarily mean that the first is the cause of the second.

So, we can see that, having used that evidence to support the conclusion, the argument is vulnerable to the criticism that "It fails to adequately address the possibility that many factors affect provincial budget deficits."

Keep.

B. It confuses a claim about the budget deficit in one province with a similar claim about the deficit in another, neighboring province.

This choice is incorrect because the argument simply doesn't do what it describes.

After all, the argument doesn't make a similar claim about the other province: it doesn't make the claim that a measure to allow union bargaining in the other province probably will, in the future, reduce the other provinces's deficit.

Also, the argument doesn't confuse any claims with each other.

Eliminate.

C. It overlooks the possibility that the budget deficit of the province immediately to the north was higher before that province allowed the union bargaining.

This choice is counterfactual.

After all, the argument actually says, "The province immediately north of ours allows such union bargaining, and its budget deficit is ... lower than it was before the province allowed union bargaining."

The stated fact that the budget deficit of province immediately to the north is lower than it was before the province allowed union bargaining means that the budget deficit of province immediately to the north was higher before it allowed union bargaining.

So, the argument clearly does not overlook "the possibility that the budget deficit of the province immediately to the north was higher before that province allowed the union bargaining."

Eliminate.

D.It takes for granted that if the measure reduces the budget deficit, then it will not increase wages of government employees.

The argument does not involve the conclusion or require the assumption that "if the measure reduces the budget deficit, then it will not increase wages of government employees."

After all, the argument works as long as the measure would reduce the deficit, regardless of what else goes on.

So, it doesn't take the idea mentioned by this choice for granted since it doesn't involve or rely on that idea.

Eliminate.
 
E. It fails to adequately distinguish a factual claim about the measure's likely consequences from a value judgment that the measure should be enacted.

This choice is incorrect because the argument simply doesn't do what it describes.

After all, the argument doesn't involve any "value judgment that the measure should be enacted." Rather, it simply makes a claim about the measure's likely consequences without getting into any value judgment.

Eliminate.

Correct answer: A­
GMAT Club Bot
Editorial: Our provinceS legislature will soon vote on a measure that [#permalink]
Moderators:
GMAT Club Verbal Expert
6923 posts
GMAT Club Verbal Expert
238 posts
CR Forum Moderator
832 posts

Powered by phpBB © phpBB Group | Emoji artwork provided by EmojiOne